LSAT and Law School Admissions Forum

Get expert LSAT preparation and law school admissions advice from PowerScore Test Preparation.

User avatar
 Hanin Abu Amara
PowerScore Staff
  • PowerScore Staff
  • Posts: 60
  • Joined: Mar 29, 2023
|
#105317
Hi!

Great question. Usually we can't make conclusions from % about absolute numbers and vice versa without additional information. In this instance they do tell us that the way the percent is determined is connected to the absolute number.

They say " each region receives a share of federal revenues equal to the share of the total population" so the percent share is dictated by the absolute number that is the share of the population. They're telling us it's proportional.

Now they also say that Korea received a decrease in its share despite seeing an increase in population. E makes sense because if they are always re allocation shares based on % and Korva saw a decrease in its share that must mean someone else had a larger increase that some them getting a larger share.

We can use percent and numbers interchangeably here because they told us that that is how they're measuring things in the stimulus.

Hope that makes sense
User avatar
 alext44
  • Posts: 1
  • Joined: Jun 19, 2024
|
#107055
Hi,

I am a little confused by this question. I chose C over E because I thought that either Mitro or Guadar could have decreased their population while the other two increased. If that decrease was enough to offset the increase in Korva's population, we couldn't guarantee that E is correct. I have mapped out my thinking below.

If initial data shows:
Korva - 34 or 34%
Guadar - 33 or 34%
Mitro - 33 or 34%

And new data shows:
Korva - 35 ~ 37%
Guadar - 25 ~ 26%
Mitro - 35 ~ 37%

The jump in population for Mitro is about 6.1% and the jump in population for Korva is about 2.9%. This is consistent with the claim in answer choice E that Korva grew by a smaller percentage than at least one of the other regions, but inconsistent with the conclusion that Korva's share of federal revenues would decrease. Perhaps I am confused about the wording in answer choice E, is it saying that they all grew, however, either Mitro or Guadar must have grown at a higher rate than Korva, or is it leaving open the possibility that either Mitro or Guadar could have had their population decrease significantly?

I chose C because guaranteeing that both Mitro and Guadar grew at a higher rate than Korva means that Korva must have decreased their share. I've mapped out my thinking for C below.

If initial data shows:
Korva - 34 or 34%
Guadar - 33 or 34%
Mitro - 33 or 34%

And new data shows:
Korva - 35 ~ 33%
Guadar - 35 ~ 33%
Mitro - 35 ~ 33%

This shows Mitro and Guadar increasing their population by about 6.1%, Korva increasing by about 2.9%, and is consistent with answer choice C and the claim that Korva's share of the revenues decreased.

Any help in clarifying this would be appreciated!

Thanks,
Alex
User avatar
 Jeff Wren
PowerScore Staff
  • PowerScore Staff
  • Posts: 451
  • Joined: Oct 19, 2022
|
#107141
Hi Alex,

First, I want to link the full explanation for this question in case you missed it, as it should help clear up any confusion.

viewtopic.php?f=657&t=11730

To directly answer your question, your comments seem to suggest that you are approaching this question as if it were a Justify question (meaning that you would be looking for an answer that proves the conclusion in the stimulus) rather than a Must Be True question (meaning that you determine what must be true based on the information in the stimulus and then find the answer that matches this inference.)

In other words, you seem to be testing each answer to see if you can come up with a fact pattern/example that matches the answer but doesn't match the stimulus. What you want to do is determine a fact pattern/example that does match the stimulus and then compare it to the answers.

Here, we are told that Korva received a smaller share of Ditrama's revenue even though Korva's population increased. (We are also told that the revenue share is equal to the region's share of Ditrama's population.) There is only one way that this can happen. At least one of the other two regions had to increase its share of the population more than Korva. If this did not happen, then Korva would have had the largest increase in population share of the three regions, which would mean that it would have received a larger share of the revenue rather than a smaller share. If you try an example that matches the facts in the stimulus, you will see that this must happen. This is correctly described in Answer E. The fact that you can come up with a different example that matches the description of Answer E and doesn't match the stimulus is irrelevant. The description in Answer E follows from the stimulus, not the other way around.

As for Answer C, while this would explain the facts in the stimulus, this does not have to be true. We wouldn't need to have Mitro and Guardar both increase their population shares more than Korva, we just need at least one.

(To see an example with numbers that shows this, please read the full question explanation linked above.)

Get the most out of your LSAT Prep Plus subscription.

Analyze and track your performance with our Testing and Analytics Package.